This document contains a series of multiple choice questions about operations management and quality based on a textbook chapter. It addresses various topics such as types of production, organizational structure, capacity, scheduling, quality improvement teams, supply chain management and more. Each question is followed by an explanation of the correct answer. The questions assess students' understanding of key concepts and terms from the chapter.
We take content rights seriously. If you suspect this is your content, claim it here.
Available Formats
Download as DOC, PDF, TXT or read online on Scribd
0 ratings0% found this document useful (0 votes)
155 views56 pages
Om
This document contains a series of multiple choice questions about operations management and quality based on a textbook chapter. It addresses various topics such as types of production, organizational structure, capacity, scheduling, quality improvement teams, supply chain management and more. Each question is followed by an explanation of the correct answer. The questions assess students' understanding of key concepts and terms from the chapter.
We take content rights seriously. If you suspect this is your content, claim it here.
Available Formats
Download as DOC, PDF, TXT or read online on Scribd
You are on page 1/ 56
Business Essentials, 9e (Ebert/Griffin)
Chapter 7 Operations Management and Quality
1) Firms that make only tangible products are engaged in goods production. Answer: TRUE Eplanation: !oods are tangible products" while ser#ices may be either tangible or intangible. $age Re%: 1&& 'i%%iculty: Easy (b)ecti#e: *.1 +earning (utcome: 'iscuss the %actors that in%luence decisions about organi,ational structure -kill: .oncept /) 0n a low1contact system" the customer must be a part o% the system to recei#e the ser#ice. Answer: FA+-E Eplanation: An eample o% a low1contact system is mail deli#ery" where the customer need not be present to recei#e the ser#ice. $age Re%: 1*2 'i%%iculty: Easy (b)ecti#e: *./ +earning (utcome: 'iscuss the %actors that in%luence decisions about organi,ational structure -kill: .oncept 3) A %irm4s capacity depends on both how many people it employs and the number and si,e o% its %acilities. Answer: TRUE Eplanation: .apacity is the amount o% a product that a company can produce under normal conditions. $age Re%: 1*3 'i%%iculty: Easy (b)ecti#e: *.5 +earning (utcome: 'iscuss the %actors that in%luence decisions about organi,ational structure -kill: .oncept 5) 0% parts A" 6" and . must be produced this week" a detailed schedule will indicate the se7uence o% work. Answer: TRUE Eplanation: 'etailed schedules show day1to1day acti#ities that will occur in production. $age Re%: 1** 'i%%iculty: Easy (b)ecti#e: *.8 +earning (utcome: 'iscuss the %actors that in%luence decisions about organi,ational structure -kill: .oncept 1 .opyright 9 /213 $earson Education" 0nc. $ublishing as $rentice :all 8) ;uality impro#ement teams are groups o% employees %rom #arious work areas who de%ine" analy,e" and sol#e common production problems. Answer: TRUE Eplanation: ;uality impro#ement teams seek to impro#e both their own work methods and the products they make. $age Re%: 1<5 'i%%iculty: Easy (b)ecti#e: *.* +earning (utcome: 'iscuss the %actors that in%luence decisions about organi,ational structure -kill: .oncept &) 0-( =222 is a certi%ication program attesting to the %act that a %actory" laboratory" or o%%ice has impro#ed its en#ironmental per%ormance. Answer: FA+-E Eplanation: This is a description o% 0-( 15222. $age Re%: 1<5 'i%%iculty: >oderate (b)ecti#e: *.* +earning (utcome: 'iscuss the roles o% ethics and corporate responsibility in business -kill: .oncept *) Each stage in the supply chain adds #alue %or the %inal customer. Answer: TRUE Eplanation: The supply chain starts with raw1materials suppliers and continues until the product reaches the end customer. $age Re%: 1<8 'i%%iculty: Easy (b)ecti#e: *.< +earning (utcome: 'iscuss the %actors that in%luence decisions about organi,ational structure -kill: .oncept <) 6y turning raw materials into %inished goods" production creates %orm utility. Answer: TRUE Eplanation: Form utility is created when raw materials and human skills are con#erted into %inished goods and ser#ices. $age Re%: 1&* 'i%%iculty: >oderate (b)ecti#e: *./ +earning (utcome: 'iscuss the %actors that in%luence decisions about organi,ational structure -kill: .oncept / .opyright 9 /213 $earson Education" 0nc. $ublishing as $rentice :all =) ?hen a company turns out ornaments in time %or .hristmas" it creates time utility. Answer: TRUE Eplanation: Time utility is created when products are made a#ailable %or consumers when they want them. $age Re%: 1&* 'i%%iculty: >oderate (b)ecti#e: *./ +earning (utcome: 'escribe the ma)or components o% e%%ecti#e distribution -kill: .oncept 12) -er#ices are more tangible" more customi,ed" and more storable than most products. Answer: FA+-E Eplanation: -er#ices are less tangible and less storable than goods. $age Re%: 1&< 'i%%iculty: >oderate (b)ecti#e: *./ +earning (utcome: 'escribe the ma)or components o% e%%ecti#e distribution -kill: .oncept 11) !etting a haircut and ha#ing your lawn mowed are both eamples o% low1contact ser#ices. Answer: FA+-E Eplanation: !etting a haircut is a high1contact ser#ice. $age Re%: 1*2 'i%%iculty: >oderate (b)ecti#e: *./ +earning (utcome: 'iscuss the %actors that in%luence decisions about organi,ational structure -kill: .oncept 1/) A company typically selects the kind o% production used by most o% its competitors. Answer: FA+-E Eplanation: A company picks the one that best achie#es its larger business strategy. $age Re%: 1*2 'i%%iculty: >oderate (b)ecti#e: *.3 +earning (utcome: 'iscuss the %actors that in%luence decisions about organi,ational structure -kill: .oncept 13) 0n a same1steps layout" e7uipment and people are set up to produce one type o% good in a %ied se7uence o% steps and are arranged according to its production re7uirements. Answer: TRUE Eplanation: A same1steps layout allows %or the e%%icient use o% an assembly line" where production operations are set up along a product1%low line. $age Re%: 1*8 'i%%iculty: >oderate (b)ecti#e: *.5 +earning (utcome: 'iscuss the %actors that in%luence decisions about organi,ational structure -kill: .oncept 3 .opyright 9 /213 $earson Education" 0nc. $ublishing as $rentice :all 15) An assembly line is an eample o% a custom1products layout. Answer: FA+-E Eplanation: An assembly line is a same1steps layout. $age Re%: 1*5 'i%%iculty: >oderate (b)ecti#e: *.5 +earning (utcome: 'iscuss the %actors that in%luence decisions about organi,ational structure -kill: .oncept 18) A process %lowchart identi%ies the se7uence o% production acti#ities" mo#ements o% materials" and work per%ormed at each stage o% the process. Answer: TRUE Eplanation: A process %lowchart can be analy,ed to isolate waste%ul acti#ities" sources o% delay" and other ine%%iciencies. $age Re%: 1*& 'i%%iculty: >oderate (b)ecti#e: *.5 +earning (utcome: 'iscuss the %actors that in%luence decisions about organi,ational structure -kill: .oncept 1&) -ta%% schedules identi%y which products will be produced and when. Answer: FA+-E Eplanation: -ta%% schedules" in general" speci%y assigned working times in upcoming days@ perhaps %or as many as 32 days or more@%or each employee on each work shi%t. $age Re%: 1** 'i%%iculty: >oderate (b)ecti#e: *.8 +earning (utcome: 'iscuss the %actors that in%luence decisions about organi,ational structure -kill: .oncept 1*) >aterials management in#ol#es the %low o% materials inside o% the production %acility" while distribution management in#ol#es the %low o% %inished goods. Answer: FA+-E Eplanation: >aterials management is the process by which managers plan" organi,e" and control the %low o% materials %rom design through distribution o% %inished goods. $age Re%: 1<2 'i%%iculty: 'i%%icult (b)ecti#e: *.& +earning (utcome: 'iscuss the %actors that in%luence decisions about organi,ational structure -kill: .oncept 5 .opyright 9 /213 $earson Education" 0nc. $ublishing as $rentice :all 1<) Aust1in1time BA0T) production systems normally re7uire large amounts o% Csa%ety stockC to be maintained in a %irm4s warehouse. Answer: FA+-E Eplanation: A0T production reduces in#entory le#els by bringing needed materials together only at the precise time they are needed %or production andDor shipment to the customer. $age Re%: 1<2 'i%%iculty: >oderate (b)ecti#e: *.& +earning (utcome: 'iscuss the %actors that in%luence decisions about organi,ational structure -kill: .oncept 1=) Aapanese companies adapted the concept o% 7uality circles %rom 7uality impro#ement teams used by U.-. companies. Answer: FA+-E Eplanation: ;uality impro#ement teams are patterned a%ter the Aapanese concept o% 7uality circles. $age Re%: 1<5 'i%%iculty: >oderate (b)ecti#e: *.* +earning (utcome: 'iscuss the %actors that in%luence decisions about organi,ational structure -kill: .oncept /2) 0-( =222 is a certi%ication program attesting that a %actory" laboratory" or o%%ice has met the rigorous re7uirements set by the 0nternational (rgani,ation %or -tandardi,ation. Answer: TRUE Eplanation: >ore than 1&2 countries ha#e adopted 0-( =222 as a national standard. $age Re%: 1<5 'i%%iculty: >oderate (b)ecti#e: *.* +earning (utcome: 'iscuss the roles o% ethics and corporate responsibility in business -kill: .oncept /1) The goal o% supply chain management is better o#erall %low in the system. Answer: TRUE Eplanation: A%ter se#eral companies work to impro#e the %low o% materials through the system" the ultimate reward is better #alue %or customers. $age Re%: 1<& 'i%%iculty: >oderate (b)ecti#e: *.< +earning (utcome: 'iscuss the %actors that in%luence decisions about organi,ational structure -kill: .oncept 8 .opyright 9 /213 $earson Education" 0nc. $ublishing as $rentice :all //) -uccess%ul companies" such as 3> and FedE" use the same operations strategy. Answer: FA+-E Eplanation: .ompanies adopt the kind o% production that achie#es their larger business strategy. $age Re%: 1*1 'i%%iculty: >oderate (b)ecti#e: *.3 +earning (utcome: 'iscuss the %actors that in%luence decisions about organi,ational structure -kill: .oncept /3) (perations capability is a special ability that production does especially well to outper%orm the competition. Answer: TRUE Eplanation: The chosen operations capability should be compatible with the o#erall business strategy. $age Re%: 1*2 'i%%iculty: >oderate (b)ecti#e: *.3 +earning (utcome: 'iscuss the %actors that in%luence decisions about organi,ational structure -kill: .oncept /5) E#en ecellent %irms %ocus on one competence at a time. Answer: FA+-E Eplanation: Ecellent %irms learn" o#er time" how to achie#e more than )ust one competence. $age Re%: 1*/ 'i%%iculty: >oderate (b)ecti#e: *.3 +earning (utcome: 'iscuss the %actors that in%luence decisions about organi,ational structure -kill: .oncept /8) According to the tet" to ensure that truckloads o% merchandise %low 7uickly to stores" ?al1 >art distribution centers are located near the hundreds o% stores that they supply" not near the companies that supply them. Answer: TRUE Eplanation: At ?al1>art" managers o% the company4s huge distribution centers regard ?al1 >art outlets as their customers. $age Re%: 1*5 'i%%iculty: >oderate (b)ecti#e: *.5 +earning (utcome: 'escribe the ma)or components o% e%%ecti#e distribution -kill: .oncept & .opyright 9 /213 $earson Education" 0nc. $ublishing as $rentice :all /&) A detailed schedule will indicate how many employees will be working on a gi#en shi%t. Answer: FA+-E Eplanation: A detailed schedule shows day1to1day acti#ities that will occur in production. A sta%% schedule identi%ies how many employees will be working. $age Re%: 1** 'i%%iculty: >oderate (b)ecti#e: *.8 +earning (utcome: 'iscuss the %actors that in%luence decisions about organi,ational structure -kill: .oncept /*) (perations control includes materials management and 7uality control. Answer: TRUE Eplanation: 6oth acti#ities are a part o% operations control and ensure that schedules are met and products deli#ered" both in 7uantity and in 7uality. $age Re%: 1*= 'i%%iculty: >oderate (b)ecti#e: *.& +earning (utcome: 'iscuss the %actors that in%luence decisions about organi,ational structure -kill: .oncept /<) For manu%acturing %irms" typical materials costs make up 82 to *8 percent o% total product costs. Answer: TRUE Eplanation: >aterials stakes are high %or ser#ice %irms as well. $age Re%: 1<2 'i%%iculty: >oderate (b)ecti#e: *.& +earning (utcome: 'iscuss strategies %or setting and ad)usting prices -kill: .oncept /=) The biggest challenge o% T;> is moti#ating employees throughout the company to achie#e 7uality goals. Answer: TRUE Eplanation: The backbone o% T;> is moti#ating employees throughout the company to achie#e 7uality goals. +eaders o% the 7uality mo#ement use #arious methods and resources to %oster a 7uality %ocusE when those e%%orts succeed" employees will ultimately accept 7uality ownership. $age Re%: 1<3 'i%%iculty: >oderate (b)ecti#e: *.* +earning (utcome: 'iscuss the %actors that a%%ect moti#ation and beha#ior in the workplace -kill: .oncept * .opyright 9 /213 $earson Education" 0nc. $ublishing as $rentice :all 32) >anagers o% a restaurant eat at a competing restaurant in order to identi%y desirable impro#ements in their own operationsE this illustrates 7uality control. Answer: FA+-E Eplanation: .ompetiti#e product analysis is a process by which a company analy,es a competitor4s products to identi%y desirable impro#ements. $age Re%: 1</ 'i%%iculty: >oderate (b)ecti#e: *.* +earning (utcome: 'escribe the skills and %unctions o% management -kill: .oncept 31) Falue1added analysis might %ocus on both the elimination o% waste and cost minimi,ation. Answer: TRUE Eplanation: Falue1added analysis re%ers to the e#aluation o% all work acti#ities" material %lows" and paperwork to determine the #alue that they add %or customersE it o%ten re#eals waste%ul or unnecessary acti#ities that can be eliminated without )eopardi,ing customer ser#ice. $age Re%: 1<3 'i%%iculty: >oderate (b)ecti#e: *.* +earning (utcome: 'escribe the skills and %unctions o% management -kill: .oncept 3/) The sales department relies on the engineering department to send samples and 7uotes to prospecti#e customers on a timely basisE the sales department may be considered the engineering department4s internal customer. Answer: TRUE Eplanation: .ustomers are both internal and eternal. $age Re%: 1<5 'i%%iculty: >oderate (b)ecti#e: *.* +earning (utcome: 'iscuss the %actors that in%luence decisions about organi,ational structure -kill: .oncept 33) 0-( 15222 may result in a recycling program at a company. Answer: TRUE Eplanation: 0-( 15222 certi%ies impro#ements in en#ironmental per%ormance" etending the 0-( approach into the arena o% en#ironmental protection and ha,ardous waste management. $age Re%: 1<5 'i%%iculty: >oderate (b)ecti#e: *.* +earning (utcome: 'iscuss the roles o% ethics and corporate responsibility in business -kill: .oncept < .opyright 9 /213 $earson Education" 0nc. $ublishing as $rentice :all 35) A #alue chain includes an entire network o% %irms" beginning with suppliers and ending when production is complete. Answer: FA+-E Eplanation: A #alue chain" also known as a supply chain" includes the %low o% in%ormation" materials" and ser#ices that starts with raw1materials suppliers and continues adding #alue through other stages in the operations process until the product reaches the end customer. $age Re%: 1<8 'i%%iculty: 'i%%icult (b)ecti#e: *.< +earning (utcome: 'iscuss the %actors that in%luence decisions about organi,ational structure -kill: .oncept 38) 6ecause supply chain strategy is based on the collecti#e e%%ort o% a number o% %irms" no one %irm in the chain gains a competiti#e ad#antage. Answer: FA+-E Eplanation: -upply chain strategy is based on the idea that members o% the chain will gain competiti#e ad#antage by working as a coordinated unit. $age Re%: 1<& 'i%%iculty: >oderate (b)ecti#e: *.< +earning (utcome: 'iscuss the %actors that in%luence decisions about organi,ational structure -kill: .oncept 3&) ?hich term re%ers to all the acti#ities in#ol#ed in making products@goods and ser#ices@%or customersG A) operations 6) processes .) planning ') scheduling E) per%ormance Answer: A Eplanation: A) C(perationsC and CproductionC re%er to the same acti#ities. $age Re%: 1&& 'i%%iculty: Easy (b)ecti#e: *.1 +earning (utcome: 'iscuss the %actors that in%luence decisions about organi,ational structure -kill: .oncept = .opyright 9 /213 $earson Education" 0nc. $ublishing as $rentice :all 3*) ?hich o% the %ollowing is the ability o% a product to satis%y a human want or needG A) utility 6) relati#e ad#antage .) content ') satis%action E) capacity Answer: A Eplanation: A) $roduction adds customer #alue by pro#iding utility. $age Re%: 1&& 'i%%iculty: Easy (b)ecti#e: *./ +earning (utcome: 'iscuss the roles o% ethics and corporate responsibility in business -kill: .oncept 3<) ?hich type o% utility is created when a company makes products a#ailable where consumers want themG A) time 6) place .) possession ') %orm E) operations Answer: 6 Eplanation: 6) An eample o% place utility is a theater showing a wide selection o% mo#ies in a popular shopping mall. $age Re%: 1&* 'i%%iculty: Easy (b)ecti#e: *./ +earning (utcome: 'iscuss the roles o% ethics and corporate responsibility in business -kill: .oncept 3=) ?hich term re%ers to a set o% methods and technologies used to produce a good or ser#iceG A) contingency plan 6) production episode .) reduction ') methods plan E) operations process Answer: E Eplanation: E) 6anks use document shredding and data encryption to protect con%idential in%ormation. 6oth are eamples o% operations processes. $age Re%: 1&= 'i%%iculty: Easy (b)ecti#e: *./ +earning (utcome: 'iscuss the %actors that in%luence decisions about organi,ational structure -kill: .oncept 12 .opyright 9 /213 $earson Education" 0nc. $ublishing as $rentice :all 52) 0n which o% the %ollowing is the customer part o% the system during ser#ice deli#eryG A) a high1contact system 6) a low1contact system .) a customer system ') a 7uality system E) a utility system Answer: A Eplanation: A) A manicure is an eample o% a high1contact system. $age Re%: 1*2 'i%%iculty: Easy (b)ecti#e: *./ +earning (utcome: 'iscuss the %actors that in%luence decisions about organi,ational structure -kill: .oncept 51) ?hat term describes a special ability that production does especially well to outper%orm the competitionG A) supply chain management 6) total 7uality management .) process engineering ') operations capability E) #alue1added analysis Answer: ' Eplanation: ') A company chooses an operation capability that is compatible with its o#erall business strategy. $age Re%: 1*2 'i%%iculty: Easy (b)ecti#e: *.3 +earning (utcome: 'iscuss the %actors that in%luence decisions about organi,ational structure -kill: .oncept 5/) ?hich o% the %ollowing is the amount o% a product that a company can produce under normal working conditions G A) capacity 6) output .) e%%iciency ') burden E) per%ormance Answer: A Eplanation: A) A %irm4s capacity depends on how many people it employs and the number and si,e o% its %acilities. $age Re%: 1*3 'i%%iculty: Easy (b)ecti#e: *.5 +earning (utcome: 'iscuss the %actors that in%luence decisions about organi,ational structure -kill: .oncept 11 .opyright 9 /213 $earson Education" 0nc. $ublishing as $rentice :all 53) E7uipment and people are grouped according to %unction in which type o% production layoutG A) cellular 6) same1steps .) custom1products ') supply1chain E) ser#ice Answer: . Eplanation: .) A custom1products layout is well suited to make1to1order shops. $age Re%: 1*5 'i%%iculty: Easy (b)ecti#e: *.5 +earning (utcome: 'iscuss the %actors that in%luence decisions about organi,ational structure -kill: .oncept 55) Thousands o% empty )ugs mo#e down a con#eyor belt at .olgate1$almoli#e be%ore stopping to be %illed and mo#ing on to be labeled. ?hat type o% layout is utili,edG A) supply1chain 6) custom1products .) hybrid ') %ied1position E) same1steps Answer: E Eplanation: E) An assembly line is a same1steps layout. $age Re%: 1*8 'i%%iculty: Easy (b)ecti#e: *.5 +earning (utcome: 'iscuss the %actors that in%luence decisions about organi,ational structure -kill: Application 58) ?hich term re%ers to how well a product does what it is supposed to doG A) capacity 6) consistency .) reliability ') per%ormance E) 7uality control Answer: ' Eplanation: ') The per%ormance o% a %ast1%ood meal might be )udged by whether it tastes good and is reasonably %illing. $age Re%: 1*8 'i%%iculty: Easy (b)ecti#e: *.5 +earning (utcome: 'iscuss the roles o% ethics and corporate responsibility in business -kill: .oncept 1/ .opyright 9 /213 $earson Education" 0nc. $ublishing as $rentice :all 5&) The sameness o% product 7uality %rom unit to unit is re%erred to by which termG A) utility 6) reliability .) per%ormance ') consistency E) 7uality control Answer: ' Eplanation: ') :igh consistency at its many locations has made .ourtyard by >arriott a leader in the lodging industry. $age Re%: 1*8 'i%%iculty: Easy (b)ecti#e: *.5 +earning (utcome: 'iscuss the roles o% ethics and corporate responsibility in business -kill: .oncept 5*) ?hich o% the %ollowing is a scheduling tool that breaks down large pro)ects into steps to be per%ormed and speci%ies the time re7uired to per%orm each oneG A) >yers graphic 6) critical path chart .) !antt chart ') process analysis graph E) master schedule Answer: . Eplanation: .) A pro)ect manager uses a !antt chart to keep the pro)ect mo#ing on schedule. $age Re%: 1*< 'i%%iculty: Easy (b)ecti#e: *.8 +earning (utcome: 'iscuss the %actors that in%luence decisions about organi,ational structure -kill: .oncept 5<) ?hich type o% system is designed %or smooth production %low to a#oid ine%%iciencies" eliminate unnecessary in#entories" and continuously impro#e production processesG A) 7uality system 6) lean system .) managed system ') production system E) %leible system Answer: 6 Eplanation: 6) +ean production systems were pioneered by Toyota. $age Re%: 1<2 'i%%iculty: Easy (b)ecti#e: *.& +earning (utcome: 'iscuss the %actors that in%luence decisions about organi,ational structure -kill: .oncept 13 .opyright 9 /213 $earson Education" 0nc. $ublishing as $rentice :all 5=) ?hich o% the %ollowing is H(T a ma)or ob)ecti#e o% A0T productionG A) increasing stop1and1go production 6) reducing goods in process .) complying with schedules ') eliminating disruptions by continuous impro#ement E) reducing in#entory le#els Answer: A Eplanation: A) A0T production replaces stop1and1go production with smooth mo#ement. $age Re%: 1<2 'i%%iculty: Easy (b)ecti#e: *.& +earning (utcome: 'iscuss the %actors that in%luence decisions about organi,ational structure -kill: .ritical Thinking 82) ?hich term re%ers to the ac7uisition o% the raw materials a company needs to produce its productsG A) de#elopment 6) purchasing .) goods control ') 7uality control E) wholesaling Answer: 6 Eplanation: 6) >ost large %irms ha#e purchasing departments to buy proper ser#ices and materials in the amounts needed. $age Re%: 1<2 'i%%iculty: Easy (b)ecti#e: *.& +earning (utcome: 'iscuss the %actors that in%luence decisions about organi,ational structure -kill: .oncept 81) ?hich o% the %ollowing re%ers to the recei#ing" storing" handling" and counting o% all raw materials" partly %inished goods" and %inished goodsG A) materials handling 6) in#entory control .) 7uality control ') wholesaling E) distribution control Answer: 6 Eplanation: 6) 0n#entory control ensures that enough materials in#entories are a#ailable to meet production schedules" while at the same time a#oiding epensi#e ecess in#entories. $age Re%: 1<2 'i%%iculty: Easy (b)ecti#e: *.& +earning (utcome: 'iscuss the %actors that in%luence decisions about organi,ational structure -kill: .oncept 15 .opyright 9 /213 $earson Education" 0nc. $ublishing as $rentice :all 8/) ?hich o% the %ollowing terms re%ers to all acti#ities in#ol#ed in getting 7uality products into the marketplaceG A) 7uality control 6) 7uality reliability .) total 7uality management ') per%ormance 7uality E) 7uality o#er#iew Answer: . Eplanation: .) T;> must consider all aspects o% a business" including customers" suppliers" and employees. $age Re%: 1<1 'i%%iculty: Easy (b)ecti#e: *.* +earning (utcome: 'escribe the skills and %unctions o% management -kill: .oncept 83) ?hich term describes the idea that 7uality belongs to each person who creates it while per%orming a )obG A) 7uality control 6) 7uality ownership .) 7uality circles ') total 7uality management E) 7uality impro#ement teams Answer: 6 Eplanation: 6) A goal o% total 7uality management is to moti#ate employees to accept 7uality ownership. $age Re%: 1</ 'i%%iculty: Easy (b)ecti#e: *.* +earning (utcome: 'iscuss the %actors that a%%ect moti#ation and beha#ior in the workplace -kill: .oncept 85) ?hat name is gi#en to the process by which a company analy,es another company4s product to identi%y desirable impro#ements in its own productG A) 7uality reliability analysis 6) benchmarking .) per%ormance 7uality analysis ') competiti#e product analysis E) 7uality outsourcing Answer: ' Eplanation: ') Using competiti#e analysis" %or eample" Toshiba might take apart a Iero copier and test each component. $age Re%: 1</ 'i%%iculty: Easy (b)ecti#e: *.* +earning (utcome: 'iscuss the %actors that in%luence decisions about organi,ational structure -kill: .oncept 18 .opyright 9 /213 $earson Education" 0nc. $ublishing as $rentice :all 88) ?hich o% the %ollowing is patterned a%ter the success%ul Aapanese concept o% 7uality circlesG A) supply chain management 6) 7uality impro#ement team .) total 7uality management ') 7uality ownership E) #alue1added analysis Answer: 6 Eplanation: 6) ;uality impro#ement teams organi,e their own work" select leaders" and address problems in the workplace. $age Re%: 1<5 'i%%iculty: Easy (b)ecti#e: *.* +earning (utcome: 'iscuss the %actors that in%luence decisions about organi,ational structure -kill: .oncept 8&) ?hich term re%ers to the %low o% in%ormation" materials" and ser#ices %rom raw1materials suppliers through stages in the operations process until the product reaches the end customerG A) distribution chain 6) supply chain .) ser#ice channel ') distribution channel E) per%ormance channel Answer: 6 Eplanation: 6) The term supply chain re%ers to the group o% companies and stream o% acti#ities that work together to create a product. $age Re%: 1<8 'i%%iculty: Easy (b)ecti#e: *.< +earning (utcome: 'iscuss the %actors that in%luence decisions about organi,ational structure -kill: .oncept 8*) ?hich o% the %ollowing is another term %or the supply chainG A) the distribution chain 6) the distribution network .) the #alue chain ') the supply network E) the wholesale network Answer: . Eplanation: .) Each stage in the #alue chain adds #alue %or the %inal customer. $age Re%: 1<8 'i%%iculty: Easy (b)ecti#e: *.< +earning (utcome: 'iscuss the %actors that in%luence decisions about organi,ational structure -kill: .oncept 1& .opyright 9 /213 $earson Education" 0nc. $ublishing as $rentice :all 8<) ?hich term re%ers to the strategy o% paying suppliers and distributors to per%orm certain business processes or to pro#ide needed materials or ser#icesG A) outsourcing 6) reengineering .) controlling ') scheduling E) warehousing Answer: A Eplanation: A) The decision to outsource epands supply chains. $age Re%: 1<& 'i%%iculty: Easy (b)ecti#e: *.< +earning (utcome: 'iscuss the %actors that in%luence decisions about organi,ational structure -kill: .oncept 8=) ?hen a company turns out costumes in time %or :alloween" it creates which type o% utilityG A) ownership 6) time .) %orm ') place E) #alue Answer: 6 Eplanation: 6) Time utility is created when products are made a#ailable when customers want them. $age Re%: 1&* 'i%%iculty: >oderate (b)ecti#e: *./ +earning (utcome: 'iscuss the roles o% ethics and corporate responsibility in business -kill: Application &2) ?hen a company makes products a#ailable where they are con#enient %or consumers" it creates which type o% utilityG A) time 6) %orm .) place ') possession E) leisure Answer: . Eplanation: .) A %ast1%ood restaurant located in a college union is an eample o% place utility. $age Re%: 1&* 'i%%iculty: >oderate (b)ecti#e: *./ +earning (utcome: 'iscuss the roles o% ethics and corporate responsibility in business -kill: .oncept 1* .opyright 9 /213 $earson Education" 0nc. $ublishing as $rentice :all &1) 0n a business" whose )ob is it to draw up plans to trans%orm resources into products and bring together basic resources" such as knowledge" physical materials" e7uipment" and laborG A) .E( 6) operations manager .) 7uality manager ') %loor %oreman E) supply manager Answer: 6 Eplanation: 6) (perations managers are responsible %or ensuring that operations processes create #alue and pro#ide bene%its to customers. $age Re%: 1&* 'i%%iculty: >oderate (b)ecti#e: *./ +earning (utcome: 'escribe the skills and %unctions o% management -kill: .oncept &/) ?hich term describes ser#ices which cannot be produced ahead o% timeG A) trans%ormed 6) low1contact .) unstorable ') intangible E) consistent Answer: . Eplanation: .) Unstorable ser#ices cannot be used at a later time. $age Re%: 1&= 'i%%iculty: >oderate (b)ecti#e: *./ +earning (utcome: 'iscuss the %actors that in%luence decisions about organi,ational structure -kill: .oncept &3) ?hich o% the %ollowing would be considered a low1contact ser#iceG A) surgery 6) haircut .) massage ') electric power E) piano lesson Answer: ' Eplanation: ') All o% the other choices are considered high1contact ser#ices in which the consumer must be present during the ser#ice transaction. $age Re%: 1*2 'i%%iculty: >oderate (b)ecti#e: *./ +earning (utcome: 'iscuss the %actors that in%luence decisions about organi,ational structure -kill: Application 1< .opyright 9 /213 $earson Education" 0nc. $ublishing as $rentice :all &5) The check1processing operations at your bank would be considered to be which type o% systemG A) high1contact 6) low1contact .) con#ersion ') synthetic E) per%ormance Answer: 6 Eplanation: 6) .heck1processing operations can be completed without the consumer being present. $age Re%: 1*2 'i%%iculty: >oderate (b)ecti#e: *./ +earning (utcome: 'iscuss the %actors that in%luence decisions about organi,ational structure -kill: Application &8) ?hich o% the %ollowing would be considered a high1contact systemG A) gas company 6) electric company .) lawn care company ') barber shop E) postal deli#ery Answer: ' Eplanation: ') A barber4s ser#ices re7uire that the consumer be present during the ser#ice transaction. $age Re%: 1*2 'i%%iculty: >oderate AA.-6: Analytic skills (b)ecti#e: *./ +earning (utcome: 'iscuss the %actors that in%luence decisions about organi,ational structure -kill: .ritical Thinking &&) -a#e1A1+ot grocery stores use which type o% strategy %or attracting customersG A) %leibility 6) 7uality .) low1cost ') dependability E) consistency Answer: . Eplanation: .) -a#e1A1+ot o%%ers items at sa#ings up to 52 percent less than con#entional %ood chains. $age Re%: 1*1 'i%%iculty: >oderate (b)ecti#e: *.5 +earning (utcome: 'iscuss the %actors that in%luence decisions about organi,ational structure -kill: Application 1= .opyright 9 /213 $earson Education" 0nc. $ublishing as $rentice :all &*) 3> uses which strategy %or attracting customersG A) 7uality 6) low1cost .) dependability ') %leibility E) consistency Answer: ' Eplanation: ') The %leibility strategy at 3> emphasi,es new product de#elopment. $age Re%: 1*1 'i%%iculty: >oderate (b)ecti#e: *.5 +earning (utcome: 'iscuss the %actors that in%luence decisions about organi,ational structure -kill: Application &<) >achine" woodworking" and dry cleaning shops typically use which type o% layoutG A) product 6) custom1products .) same1steps ') hybrid E) make1to1stock Answer: 6 Eplanation: 6) 0n a custom1products layout" machines and people are grouped by %unction in the production %acilityE custom1products layouts allow %or greater %leibility and are well suited to make1to1order shops. $age Re%: 1*5 'i%%iculty: >oderate (b)ecti#e: *.5 +earning (utcome: 'iscuss the %actors that in%luence decisions about organi,ational structure -kill: Application &=) ?hich type o% layout is designed to mo#e resources through a smooth" %ied se7uence o% stepsG A) same1steps 6) location .) custom1products ') 7uality E) make1to1order Answer: A Eplanation: A) A same1steps layout is set up to make one type o% product in a %ied se7uence and is arranged according to its production re7uirements. $age Re%: 1*8 'i%%iculty: >oderate (b)ecti#e: *.5 +earning (utcome: 'iscuss the %actors that in%luence decisions about organi,ational structure -kill: .oncept /2 .opyright 9 /213 $earson Education" 0nc. $ublishing as $rentice :all *2) Automobile" %ood1processing" and tele#ision assembly plants use which type o% layoutG A) make1to1order 6) location .) custom1products ') 7uality E) same1steps Answer: E Eplanation: E) These production processes re7uire a %ied se7uence that is arranged according to production re7uirements. $age Re%: 1*8 'i%%iculty: >oderate (b)ecti#e: *.5 +earning (utcome: 'iscuss the %actors that in%luence decisions about organi,ational structure -kill: Application *1) ?hich term is de%ined as the combination o% Ccharacteristics o% a product or ser#ice that bear on its ability to satis%y stated or implied needsCG A) 7uality 6) production .) 7uantity ') clari%ication E) capacity Answer: A Eplanation: A) -uch characteristics can include a reasonable price. $age Re%: 1*8 'i%%iculty: >oderate (b)ecti#e: *.5 +earning (utcome: 'iscuss the roles o% ethics and corporate responsibility in business -kill: .oncept */) $er%ormance re%ers to which o% the %ollowingG A) the principle that 7uality belongs to each person who creates it while per%orming a )ob 6) the consistency o% product 7uality %rom unit to unit .) how well the product does what it is supposed to do ') the process by which a company analy,es a competitor4s products to identi%y desirable impro#ement E) pro#iding #alue by making products a#ailable when customers want them Answer: . Eplanation: .) $lanning %or 7uality begins when products are being designed. Early in the process" goals are established %or both per%ormance and consistency. $age Re%: 1*8 'i%%iculty: >oderate (b)ecti#e: *.5 +earning (utcome: 'iscuss the roles o% ethics and corporate responsibility in business -kill: .oncept /1 .opyright 9 /213 $earson Education" 0nc. $ublishing as $rentice :all *3) .onsistency re%ers to which o% the %ollowingG A) the principle that 7uality belongs to each person who creates it while per%orming a )ob 6) the sameness o% product 7uality %rom unit to unit .) the sum o% all acti#ities in#ol#ed in getting high17uality products into the marketplace ') the process by which a company analy,es a competitor4s products to identi%y desirable impro#ements E) pro#iding #alue by making products a#ailable where customers want them Answer: 6 Eplanation: 6) This consistency is achie#ed by controlling %or consistent raw materials" encouraging conscientious work" and maintaining e7uipment. $age Re%: 1*8 'i%%iculty: >oderate (b)ecti#e: *.5 +earning (utcome: 'iscuss the roles o% ethics and corporate responsibility in business -kill: .oncept *5) >anagers can work to reduce waste" ine%%iciency" and poor per%ormance by eamining procedures on a step1by1step basis. ?hich term describes this processG A) ad#ance planning 6) materials management .) methods impro#ement ') 7uality planning E) 7uality ownership Answer: . Eplanation: .) A process %lowchart is sometimes used to assist in methods impro#ement. $age Re%: 1*& 'i%%iculty: >oderate (b)ecti#e: *.5 +earning (utcome: 'escribe the skills and %unctions o% management -kill: .oncept *8) +ogan Aluminum makes coils o% aluminum that it supplies to customer companies that use it to make be#erage cans. +ogan uses a schedule that speci%ies how many tons o% each type o% coil will be produced each week. ?hat is this schedule calledG A) master production schedule 6) detailed schedule .) !antt chart ') sta%% schedule E) $ERT chart Answer: A Eplanation: A) The master production schedule shows which products will be produced" and when" in upcoming time periods. $age Re%: 1** 'i%%iculty: >oderate (b)ecti#e: *.8 +earning (utcome: 'escribe the skills and %unctions o% management -kill: Application // .opyright 9 /213 $earson Education" 0nc. $ublishing as $rentice :all *&) 0n operations control" production managers monitor production per%ormance by which methodG A) comparing results with detailed plans and schedules 6) checking on each worker on the production %loor .) obser#ing acti#ities %rom a plat%orm abo#e the workers ') constantly talking with employees in#ol#ed E) per%orming spot checks o% worker per%ormance Answer: A Eplanation: A) 0% schedules or 7uality standards aren4t met" managers can take correcti#e action. $age Re%: 1*= 'i%%iculty: >oderate (b)ecti#e: *.& +earning (utcome: 'escribe the skills and %unctions o% management -kill: .oncept **) ?hich o% the %ollowing is H(T one o% the areas o% materials managementG A) transportation 6) warehousing .) purchasing ') supplier selection E) customer ser#ice Answer: E Eplanation: E) >aterials management includes supplier selection" purchasing" transportation" warehousing" and in#entory control. $age Re%: 1<2 'i%%iculty: >oderate (b)ecti#e: *.& +earning (utcome: 'escribe the skills and %unctions o% management -kill: .oncept /3 .opyright 9 /213 $earson Education" 0nc. $ublishing as $rentice :all *<) ?hat is the name %or a production system in which all the needed materials and parts arri#e at the precise moment they are re7uired %or each production stageG A) 7uality control system 6) process control system .) )ust1in1time production ') standardi,ed production E) custom1products production Answer: . Eplanation: .) A0T production brings together all needed materials at the precise moment they are re7uired %or each production stage" not be%ore" creating e%%icient responses to customer orders. $age Re%: 1<2 'i%%iculty: >oderate (b)ecti#e: *.& +earning (utcome: 'iscuss the %actors that in%luence decisions about organi,ational structure -kill: .oncept *=) ?hich o% the %ollowing is greatly reduced in a )ust1in1time production systemG A) number o% workers needed on the line 6) number o% goods in process .) number o% shi%ts necessary at the plant ') number o% separate operations on the assembly line E) number o% %orepersons needed at the plant Answer: 6 Eplanation: 6) As a result" A0T production minimi,es in#entory costs and sa#es money by replacing stop1and1go production with smooth mo#ement. $age Re%: 1<2 'i%%iculty: >oderate (b)ecti#e: *.& +earning (utcome: 'iscuss the %actors that in%luence decisions about organi,ational structure -kill: .oncept <2) ?hich o% the %ollowing is a component o% materials managementG A) promotion 6) distribution .) marketing ') warehousing E) sales Answer: ' Eplanation: ') >aterials management includes supplier selection" purchasing" transportation" warehousing" and in#entory control. $age Re%: 1<2 'i%%iculty: >oderate (b)ecti#e: *.& +earning (utcome: 'escribe the skills and %unctions o% management -kill: .oncept /5 .opyright 9 /213 $earson Education" 0nc. $ublishing as $rentice :all <1) ?hich o% the %ollowing best describes total 7uality managementG A) the principle that 7uality belongs to each person who creates it while per%orming a )ob 6) the consistency o% a product4s 7uality %rom unit to unit .) the sum o% all acti#ities in#ol#ed in getting high17uality products into the marketplace ') the process by which a company analy,es a competitor4s products to identi%y desirable impro#ements E) a certi%ication program attesting that an operations process has met rigorous re7uirements Answer: . Eplanation: .) T;> begins with leadership and a desire %or continuously impro#ing both processes and productsE it must consider all aspects o% a business" including customers" suppliers" and employees. $age Re%: 1<1 'i%%iculty: >oderate (b)ecti#e: *.* +earning (utcome: 'escribe the skills and %unctions o% management -kill: .oncept </) ?hich o% the %ollowing best describes 7uality ownershipG A) an emphasis on intangible %actors in customer satis%action 6) the consistency o% a product4s 7uality %rom unit to unit .) the sum o% all acti#ities in#ol#ed in getting high17uality products into the marketplace ') the process by which a company analy,es a competitor4s products to identi%y desirable impro#ements E) the principle that 7uality belongs to each person who creates it while per%orming a )ob Answer: E Eplanation: E) ;uality1%ocused leaders use #arious methods to %oster a 7uality %ocus among the work%orceE when these e%%orts succeed" employees will ultimately accept 7uality ownership. $age Re%: 1</ 'i%%iculty: >oderate (b)ecti#e: *.* +earning (utcome: 'iscuss the %actors that a%%ect moti#ation and beha#ior in the workplace -kill: .oncept /8 .opyright 9 /213 $earson Education" 0nc. $ublishing as $rentice :all <3) ?hich o% the %ollowing best describes competitive product analysisG A) the principle that 7uality belongs to each person who creates it while per%orming a )ob 6) the consistency o% a product4s 7uality %rom unit to unit .) the process by which a company analy,es a di%%erent company4s products to identi%y desirable impro#ements ') the sum o% all acti#ities in#ol#ed in getting a high17uality product into the marketplace. E) eamining a product to impro#e its %orm utility Answer: . Eplanation: .) This analysis helps managers decide which product %eatures are satis%actory" which %eatures should be upgraded" and which operations processes need impro#ement" %or eample. $age Re%: 1</ 'i%%iculty: >oderate (b)ecti#e: *.* +earning (utcome: 'iscuss the %actors that in%luence decisions about organi,ational structure -kill: .oncept <5) ?hen a worker at Toshiba takes apart a Iero copier and tests each component" it is engaging in what acti#ityG A) competiti#e product analysis 6) benchmarking .) total 7uality management ') 7uality reliability analysis E) #alue1added analysis Answer: A Eplanation: A) .ompetiti#e product analysis is the process by which a company analy,es a competitor4s products to identi%y desirable impro#ements. $age Re%: 1</ 'i%%iculty: >oderate AA.-6: Re%lecti#e thinking skills (b)ecti#e: *.* +earning (utcome: 'iscuss the %actors that in%luence decisions about organi,ational structure -kill: Application /& .opyright 9 /213 $earson Education" 0nc. $ublishing as $rentice :all <8) ?hich o% the %ollowing best describes #alue1added analysisG A) a T;> tool in which groups o% employees work together to impro#e 7uality 6) the process o% e#aluating all work acti#ities" materials %lows" and paperwork to determine the #alue that they create %or customers .) the process by which a company implements the best practices %rom its own past per%ormance and those o% other companies to impro#e its own products ') the process by which a company analy,es a product to identi%y possible impro#ements E) a total company commitment to 7uality management at e#ery stage o% the process Answer: 6 Eplanation: 6) Falue1added analysis o%ten re#eals waste%ul or unnecessary acti#ities that can be eliminated without )eopardi,ing customer ser#ice. $age Re%: 1<3 'i%%iculty: >oderate (b)ecti#e: *.* +earning (utcome: 'iscuss the %actors that in%luence decisions about organi,ational structure -kill: .oncept <&) Tootsie Roll 0ndustry4s corporate principle" C?e run a trim operation and continually stri#e to eliminate waste" minimi,e costs" and implement per%ormance impro#ements"C is an eample o% which o% the %ollowingG A) )ust1in1time production 6) benchmarking .) a 7uality circle ') competiti#e product analysis E) #alue1added analysis Answer: E Eplanation: E) Tootsie Roll 0ndustry e#aluates all work acti#ities" material %lows" and paperwork to determine the #alue that they add %or customers. $age Re%: 1<3 'i%%iculty: >oderate AA.-6: Re%lecti#e thinking skills (b)ecti#e: *.* +earning (utcome: 'iscuss the %actors that in%luence decisions about organi,ational structure -kill: Application /* .opyright 9 /213 $earson Education" 0nc. $ublishing as $rentice :all <*) ?hich term re%ers to collaborati#e groups o% employees %rom #arious work areas who meet regularly to de%ine" analy,e" and sol#e common production problemsG A) 7uality impro#ement teams 6) 7uality assurance teams .) 7uality control teams ') T;> teams E) 7uality re#iew teams Answer: A Eplanation: A) ;uality impro#ement teams organi,e their own work" select leaders" and address problems in the workplace. $age Re%: 1<5 'i%%iculty: >oderate (b)ecti#e: *.* +earning (utcome: 'iscuss the %actors that in%luence decisions about organi,ational structure -kill: .oncept <<) ?hich o% the %ollowing best describes 0-( =222G A) the concept that all employees are #aluable contributors to a %irm4s business" and should be entrusted with decisions regarding their work 6) the redesigning o% business processes to impro#e per%ormance" 7uality" and producti#ity .) a program certi%ying that a %actory" laboratory" or o%%ice has met the 7uality management standards o% the 0nternational (rgani,ation %or -tandardi,ation ') a certi%ication program attesting to the %act that a %actory" laboratory" or o%%ice has impro#ed en#ironmental per%ormance E) the process o% e#aluating all work acti#ities to determine the #alue they add %or customers Answer: . Eplanation: .) 0-( =222 certi%ies that a %actory" laboratory" or o%%ice has met the 7uality management standards o% 0-(E these standards are now regarded as a national standard %or more than 1&2 countries. $age Re%: 1<5 'i%%iculty: >oderate (b)ecti#e: *.* +earning (utcome: 'iscuss the roles o% ethics and corporate responsibility in business -kill: .oncept /< .opyright 9 /213 $earson Education" 0nc. $ublishing as $rentice :all <=) ?hich program certi%ies impro#ements in en#ironmental per%ormanceG A) 0-( =222 6) 0-( =222:/222 .) 0-( 15222 ') T;> /221 E) T;> /21/ Answer: . Eplanation: .) Etending the 0-( approach into the arena o% en#ironmental protection and ha,ardous waste management" 0-( 15222 re7uires a %irm to de#elop an en#ironmental management system. $age Re%: 1<5 'i%%iculty: >oderate (b)ecti#e: *.* +earning (utcome: 'iscuss the roles o% ethics and corporate responsibility in business -kill: .oncept =2) ?hich o% the %ollowing best describes 0-( 15222G A) the concept that all employees are #aluable contributors to a %irm4s business and should be entrusted with decisions regarding their work 6) a certi%ication program attesting to the %act that a %actory" laboratory" or o%%ice has impro#ed en#ironmental per%ormance .) a certi%ication program attesting to the %act that a %actory" laboratory" or o%%ice has met the 7uality management standards o% the 0nternational (rgani,ation %or -tandardi,ation ') a standardi,ed method o% e#aluating a company4s greenhouse gas emissions E) a certi%ication program attesting to the %act that a %actory" laboratory" or o%%ice is operating under %ree1trade principles Answer: 6 Eplanation: 6) 0-( 15222 re7uires a %irm to de#elop an en#ironmental management system" documenting how the company has acted to impro#e its per%ormance in using resources and in managing pollution. $age Re%: 1<5 'i%%iculty: >oderate (b)ecti#e: *.* +earning (utcome: 'iscuss the roles o% ethics and corporate responsibility in business -kill: .oncept /= .opyright 9 /213 $earson Education" 0nc. $ublishing as $rentice :all =1) Recei#ing and storing materials" billing patients %or treatment" and %illing customer orders %rom 0nternet sales are eamples o% which acti#ityG A) %unction 6) techni7ue .) process ') benchmark E) 7uality ownership Answer: . Eplanation: .) E#ery business consists o% processes" which are acti#ities that are per%ormed regularly and routinely in conducting business. $age Re%: 1<8 'i%%iculty: >oderate (b)ecti#e: *.* +earning (utcome: 'iscuss the %actors that in%luence decisions about organi,ational structure -kill: Application =/) ?hich term re%ers to the %undamental rethinking and radical redesign o% a business acti#ity to achie#e dramatic impro#ements in per%ormanceG A) supply chain management 6) business process reengineering .) total 7uality management ') 7uality ownership E) 7uality control Answer: 6 Eplanation: 6) Rethinking the production steps by starting %rom scratch has allowed dramatic impro#ements in cost" 7uality" ser#ice" and speed. $age Re%: 1<8 'i%%iculty: >oderate (b)ecti#e: *.* +earning (utcome: 'iscuss the %actors that in%luence decisions about organi,ational structure -kill: .oncept 32 .opyright 9 /213 $earson Education" 0nc. $ublishing as $rentice :all =3) ?hich o% the %ollowing best describes business process reengineeringG A) the redesigning o% business acti#ities to impro#e per%ormance" 7uality" and producti#ity 6) a program certi%ying that a %actory" laboratory" or o%%ice has met the 7uality management standards o% the 0nternational (rgani,ation %or -tandardi,ation .) the principle o% looking at the supply chain as a whole in order to impro#e the o#erall %low through the system ') a program certi%ying that a %actory" laboratory" or o%%ice has de#eloped pollution1control standards E) the process by which a company analy,es a di%%erent company4s products to identi%y desirable impro#ements Answer: A Eplanation: A) 6usiness process reengineering %ocuses on impro#ing a business process" rethinking each o% its steps %rom start to %inish. $age Re%: 1<8 'i%%iculty: >oderate (b)ecti#e: *.* +earning (utcome: 'iscuss the %actors that in%luence decisions about organi,ational structure -kill: .oncept =5) ?hich o% the %ollowing best describes supply chain managementG A) the principle o% impro#ing the supply chain by %ocusing on the slowest step in the chain 6) a limitation on the number o% suppliers allowed to supply a particular company .) complete mo#ement o% raw materials throughout a manu%acturing or ser#ice %acility ') a cost1reduction program in which wholesalers and retailers are eliminated in an e%%ort to entice consumers to purchase directly %rom manu%acturers E) the principle o% looking at the supply chain as a whole in order to impro#e the o#erall %low through the system Answer: E Eplanation: E) 0n addition" because customers ultimately get better #alue" supply chain management gains competiti#e ad#antage %or each supply chain member. $age Re%: 1<& 'i%%iculty: >oderate (b)ecti#e: *.< +earning (utcome: 'escribe the skills and %unctions o% management -kill: .oncept 31 .opyright 9 /213 $earson Education" 0nc. $ublishing as $rentice :all =8) ?hen 'ell 0nc. shares in%ormation to impro#e the o#erall %low through a system composed o% companies working together" it is engaging in which o% the %ollowingG A) distribution chain management 6) total 7uality management .) supply chain management ') ser#ice channel analysis E) supplier selection analysis Answer: . Eplanation: .) >ichael 'ell4s #ision in#ol#es the concept o% impro#ing per%ormance by sharing in%ormation among supply chain membersE long1term production plans and up1to1the1 minute sales data are a#ailable to suppliers #ia the 0nternet. $age Re%: 1<& 'i%%iculty: >oderate (b)ecti#e: *.< +earning (utcome: 'escribe the skills and %unctions o% management -kill: Application =&) ?hen Ford assembles parts into a Ford Eplorer" it creates which type o% utilityG A) time 6) %orm .) possession ') place E) ownership Answer: 6 Eplanation: 6) Form utility is created when raw materials are con#erted into %inished products. $age Re%: 1&* 'i%%iculty: >oderate (b)ecti#e: *./ +earning (utcome: 'iscuss the %actors that in%luence decisions about organi,ational structure -kill: Application =*) ?hich operations competence has Toyota gi#en greatest credit %or winning orders in the marketplaceG A) 7uality 6) low cost .) %leibility ') dependability E) eclusi#ity Answer: A Eplanation: A) Toyota has %ocused on creating reliable cars with an appealing %it and %inish" and assuring that customer epectations are met or eceeded in order to build the company4s 7uality competence. $age Re%: 1*1 'i%%iculty: >oderate (b)ecti#e: *.3 +earning (utcome: 'iscuss the %actors that in%luence decisions about organi,ational structure -kill: Application 3/ .opyright 9 /213 $earson Education" 0nc. $ublishing as $rentice :all =<) ?hich operations competence has FedE gi#en greatest credit %or winning orders Bbusiness) in the marketplaceG A) 7uality 6) low cost .) %leibility ') dependability E) accessibility Answer: ' Eplanation: ') FedE wants to assure that e#ery deli#ery is %ast and on time" as promised. $age Re%: 1*1 'i%%iculty: >oderate (b)ecti#e: *.3 +earning (utcome: 'iscuss the %actors that in%luence decisions about organi,ational structure -kill: Application ==) Eamining step1by1step procedures to reduce ine%%iciency most centrally in#ol#es which approachG A) operations scheduling 6) per%ormance analysis .) #alue1added analysis ') methods impro#ement E) 7uality ownership Answer: ' Eplanation: ') >ethods impro#ement has been used to streamline the traditional checkout method at hotels. $age Re%: 1*& 'i%%iculty: 'i%%icult (b)ecti#e: *.5 +earning (utcome: 'escribe the skills and %unctions o% management -kill: .oncept 122) ?hich scheduling tool shows the necessary se7uence o% acti#ities in a pro)ect and identi%ies the critical pathG A) !antt chart 6) process %lowchart .) detailed schedule ') master production schedule E) $ERT chart Answer: E Eplanation: E) The critical path is the most time1consuming set o% acti#ities in a pro)ect. $age Re%: 1** 'i%%iculty: 'i%%icult (b)ecti#e: *.8 +earning (utcome: 'iscuss the %actors that in%luence decisions about organi,ational structure -kill: .oncept 33 .opyright 9 /213 $earson Education" 0nc. $ublishing as $rentice :all 121) ?hen :ewlett1$ackard simpli%ied its contracts and reduced them %rom /2 pages to as %ew as /" it was engaging in which acti#ityG A) #alue1added analysis 6) benchmarking .) getting closer to the customer ') supply chain management E) %ollow1up Answer: A Eplanation: A) Falue1added analysis re%ers to the e#aluation o% all work acti#ities" materials %lows" and paperwork to determine the #alue that they add %or customers. $age Re%: 1<3 'i%%iculty: 'i%%icult (b)ecti#e: *.* +earning (utcome: 'escribe the skills and %unctions o% management -kill: Application 12/) 'e%ine the term production or operations. Answer: The term operations Bor production) re%ers to all the acti#ities in#ol#ed in making products@goods and ser#ices@%or customers. Eplanation: A key di%%erence between goods and ser#ices operations is the customer4s in#ol#ement in the latter. $age Re%: 1&& 'i%%iculty: >oderate (b)ecti#e: *.1 +earning (utcome: 'iscuss the %actors that in%luence decisions about organi,ational structure -kill: .oncept 123) ?hat is an operations capabilityG Answer: (perations capability re%ers to the acti#ity or process that production does especially well. Each company4s operations capability matches up with its business strategy so that the %irm4s acti#ities" %rom top to bottom" are %ocused in a particular direction. Eplanation: For eample" the operations capability o% FedE is dependability. $age Re%: 1*2 'i%%iculty: >oderate (b)ecti#e: *.3 +earning (utcome: 'iscuss the %actors that in%luence decisions about organi,ational structure -kill: .oncept 35 .opyright 9 /213 $earson Education" 0nc. $ublishing as $rentice :all 125) Eplain competiti#e product analysis. Answer: .ompetiti#e product analysis is a process by which a company analy,es a competitor4s products to identi%y desirable impro#ements in its own products. Eplanation: For eample" a small business owner might study a competitor4s ?eb site to %ind ways to impro#e her own site. $age Re%: 1</ 'i%%iculty: >oderate (b)ecti#e: *.* +earning (utcome: 'iscuss the %actors that in%luence decisions about organi,ational structure -kill: .oncept 128) 'i%%erentiate between goods production and ser#ice operations. Answer: !oods production in#ol#es tangible products" such as radios" newspapers" buses" and tetbooks. Through ser#ice operations" %irms produce tangible and intangible ser#ice products such as entertainment" transportation" and education. Eplanation: !eneral Electric is a company that produces goods and pro#ides ser#ices. $age Re%: 1&& 'i%%iculty: >oderate (b)ecti#e: *.1 +earning (utcome: 'iscuss the %actors that in%luence decisions about organi,ational structure -kill: .oncept 12&) 'i%%erentiate between high1 and low1contact processes. !i#e an eample o% each. Answer: $rocesses may be classi%ied according to the etent o% customer contact as either high1 or low1contact processes. 0n a high1contact process" the customer must be a part o% the system. Eamples o% high1contact processes include public transportation" medical ser#ices" and most legal ser#ices. ?ith a low1contact process" customers need not be a part o% the system to recei#e the ser#ice. Eamples include check1processing operations at the bank and lawn1care ser#ices. Eplanation: The manager o% a high1contact process must be more concerned with the customer eperience. $age Re%: 1*2 'i%%iculty: >oderate (b)ecti#e: *./ +earning (utcome: 'iscuss the %actors that in%luence decisions about organi,ational structure -kill: .oncept 12*) ?hat is a !antt chartG Answer: A !antt chart is a production schedule that breaks down large pro)ects into steps to be per%ormed and speci%ies the time re7uired to per%orm each step. Eplanation: The pro)ect manager checks the progress against the time scale and may need to add workers i% the pro)ect %alls behind schedule. $age Re%: 1*< 'i%%iculty: >oderate (b)ecti#e: *.8 +earning (utcome: 'iscuss the %actors that in%luence decisions about organi,ational structure -kill: .oncept 38 .opyright 9 /213 $earson Education" 0nc. $ublishing as $rentice :all 12<) ?hat is 7uality ownershipG Answer: ;uality ownership is the principle o% total 7uality management that holds that 7uality belongs to each person who creates it while per%orming a )ob. Eplanation: A goal o% total 7uality management is moti#ating employees to accept 7uality ownership. $age Re%: 1</ 'i%%iculty: >oderate (b)ecti#e: *.* +earning (utcome: 'iscuss the %actors that a%%ect moti#ation and beha#ior in the workplace -kill: .oncept 12=) Eplain what is meant by the term utility. ?hat type o% utility is created when a pharmacy changes its e#ening schedule to remain open %or an etra hourG Answer: Utility is a product4s ability to satis%y a human want or need. The pharmacy creates time utility by being open at a con#enient time %or customers. Eplanation: The three types o% utility are %orm utility" place utility" and time utility. $age Re%: 1&& 'i%%iculty: 'i%%icult (b)ecti#e: *./ +earning (utcome: 'iscuss the roles o% ethics and corporate responsibility in business -kill: Application 112) ?hy does each company4s operations capability match up with its business strategyG Answer: Each company4s operations capability matches up with its business strategy so that the %irm4s acti#ities " %rom top to bottom" are %ocused in a particular direction. Aligning operations capability with business strategy helps to impro#e pro%itability. Eplanation: For eample" FedE %ocuses on dependability which has made it a leader with business customers. $age Re%: 1*211*1 'i%%iculty: 'i%%icult (b)ecti#e: *.3 +earning (utcome: 'iscuss the %actors that in%luence decisions about organi,ational structure -kill: .oncept 111) 'escribe two alternati#es %or production %acility layouts. Answer: 0n a custom1products layout" e7uipment and people are grouped according to %unctionE in a same1steps layout" e7uipment and people are set up to produce one type o% product in a %ied se7uence o% steps and are arranged according to production re7uirements. Eplanation: A custom1products layout is well suited %or make1to1order shops while a same1 steps layout is e%%icient %or make1to1stock operations. $age Re%: 1*511*8 'i%%iculty: 'i%%icult (b)ecti#e: *.5 +earning (utcome: 'iscuss the %actors that in%luence decisions about organi,ational structure -kill: .oncept 3& .opyright 9 /213 $earson Education" 0nc. $ublishing as $rentice :all 11/) Eplain how A0T production and lean manu%acturing are related. Answer: +ean manu%acturing is designed %or smooth production %lows that a#oid ine%%iciencies" eliminate unnecessary in#entories" and continuously impro#e production processes. Aust1in1time BA0T) production" a type o% lean system" brings together all needed materials at the precise moment they are re7uired %or each production stage. Eplanation: ?ith A0T production" disruptions are more #isible and are resol#ed more 7uickly. $age Re%: 1<2 'i%%iculty: 'i%%icult (b)ecti#e: *.& +earning (utcome: 'iscuss the %actors that in%luence decisions about organi,ational structure -kill: -ynthesis 113) 0n a T;> en#ironment" what are some steps that companies use to emphasi,e the importance o% 7ualityG Answer: To ensure high17uality goods and ser#ices" many %irms assign responsibility %or some aspects o% T;> to speci%ic departments or positions. >ore broadly" leaders o% the 7uality mo#ement use #arious methods and resources to %oster a 7uality %ocus@training" #erbal encouragement" teamwork" and tying compensation to work 7uality. Eplanation: Total 7uality management BT;>) includes all the acti#ities necessary %or getting high17uality goods and ser#ices into the marketplace. $age Re%: 1<3 'i%%iculty: 'i%%icult (b)ecti#e: *.* +earning (utcome: 'escribe the skills and %unctions o% management -kill: .oncept 115) ?hat is a supply chainG :ow can supply chain management create competiti#e ad#antageG Answer: A supply chain is the %low o% in%ormation" materials" and ser#ices that starts with raw materials suppliers and continues through other steps in the operations process until the product reaches the end consumer. -upply chain management creates better #alue %or the customer to produce a competiti#e ad#antage %or the members o% the chain. Eplanation: 'ell" 0nc. has used an inno#ati#e supply chain strategy to help lower prices and speed deli#ery o% its $.s. $age Re%: 1<811<& 'i%%iculty: 'i%%icult (b)ecti#e: *.< +earning (utcome: 'escribe the skills and %unctions o% management -kill: .oncept 3* .opyright 9 /213 $earson Education" 0nc. $ublishing as $rentice :all 118) :ow does outsourcing a%%ect global supply chain managementG Answer: (utsourcing is the strategy o% paying suppliers and distributors to per%orm certain business processes or to pro#ide needed materials or ser#ices. Arrangements %or cross1border materials %lows re7uire compliance with each country4s commerce regulations. $roduction and global transportation scheduling are coordinated with U.-. market demand so that outsourced products arri#e in the correct amount and on time without harming the manu%acturer4s image. Eplanation: (utsourcing has been a growing trend in American business. (utsourcers ha#e a greater need o% operations skills %or integration among dispersed %acilities. $age Re%: 1<&11<* 'i%%iculty: 'i%%icult AA.-6: 'ynamics o% the global economy (b)ecti#e: *.< +earning (utcome: Eplain the bene%its and challenges o% engaging in international business -kill: -ynthesis 11&) Eplain time" place" and %orm utility. Answer: Time utility is created when a company makes products a#ailable when consumers want them. Eamples may include seasonal items such as .hristmas ornaments" bathing suits" and mos7uito repellent. $lace utility is created when a company makes a product4s location con#enient %or consumers. An eample is the placement o% routinely purchased items in con#enience stores or con#eniently located discount stores. Form utility is created when a company combines materials to create products. Eplanation: A business adds customer #alue by pro#iding utility. $age Re%: 1&* 'i%%iculty: >oderate (b)ecti#e: *./ +earning (utcome: 'iscuss the roles o% ethics and corporate responsibility in business -kill: Application 11*) The tet says that Toyota has %ocused on 7uality as its operations capability. Eplain how Toyota4s operations might ha#e changed i% it had %ocused on low cost instead o% 7uality. Answer: A cost1minimi,ation %ocus would ha#e been appropriate in this case" gi#ing Toyota4s operations an altogether di%%erent %orm. $ossible changes include more outsourcing o% production" reduced ad#ertising" %ewer models" and %ewer accessories on the cars. Eplanation: (perations capability is the particular ability that production does especially well to outper%orm the competition. $age Re%: 1*1 'i%%iculty: 'i%%icult AA.-6: Analytic skills (b)ecti#e: *.3 +earning (utcome: 'iscuss the %actors that in%luence decisions about organi,ational structure -kill: Application 3< .opyright 9 /213 $earson Education" 0nc. $ublishing as $rentice :all 11<) 'escribe the %i#e most commonly used tools %or T;>. Answer: The most commonly used tools %or T;> are #alued1added analysis" 7uality impro#ement teams" getting closer to the customer" the 0-( series" and business process reengineering. Falue1added analysis re%ers to the e#aluation o% all work acti#ities" material %lows" and paperwork to determine the #alue that they add %or customers. ;uality impro#ement teams are groups o% employees %rom #arious work areas who meet regularly to de%ine" analy,e" and sol#e common production problems. !etting closer to the customer in#ol#es taking steps to know what customers want in the products they consume. The 0-( series pertains to a series o% certi%ications attesting that a %actory" laboratory" or o%%ice adheres to rigorous 7uality management re7uirements set by the 0nternational (rgani,ation %or -tandardi,ation. 6usiness process reengineering %ocuses on impro#ing a business process@rethinking each o% the process4s steps by starting %rom scratchE the process yields impro#ements as measured by cost" 7uality" speed" and ser#ice. Eplanation: T;> begins with leadership and a desire %or continuously impro#ing both processes and products. 0t must consider all aspects o% a business" including customers" suppliers" and employees. $age Re%: 1<311<8 'i%%iculty: 'i%%icult (b)ecti#e: *.* +earning (utcome: 'escribe the skills and %unctions o% management -kill: .oncept 11=) :ow are !eneral >otors and .hrysler responding to current economic conditionsG Answer: 6oth companies ha#e adopted leaner operations to simpli%y production. Eplanation: 0n a slow economy" businesses maintain pro%itability through cutting costs and impro#ing e%%iciency. $age Re%: 1</ 'i%%iculty: >oderate (b)ecti#e: *.& +earning (utcome: 'iscuss the %actors that in%luence decisions about organi,ational structure -kill: .oncept 1/2) :ow ha#e !eneral >otors and .hrysler implemented leaner operationsG Answer: A smaller number o% makes" models" and options simpli%ies product design" production" and distribution. Eplanation: !>" .hrysler" and Ford are adopting business strategies that Aapanese producers ha#e used to simpli%y production and capture a greater market share. $age Re%: 1</ 'i%%iculty: >oderate (b)ecti#e: *.& +earning (utcome: 'iscuss the %actors that in%luence decisions about organi,ational structure -kill: Application 3= .opyright 9 /213 $earson Education" 0nc. $ublishing as $rentice :all 1/1) ?hat changes can be epected in !eneral >otors and .hrysler4s distribution networks as a result o% leaner operationsG Answer: The companies4 distribution networks will be simpli%ied as some brands are eliminated. Fewer brands will reduce the number o% auto dealerships and lower distribution costs. Eplanation: !> has downsi,ed to %our core brands and .hrysler4s roster includes only three ma)or brands. $age Re%: 1</ 'i%%iculty: 'i%%icult (b)ecti#e: *.& +earning (utcome: 'escribe the ma)or components o% e%%ecti#e distribution -kill: -ynthesis 1//) 'escribe how a !antt chart and a $ERT chart are similar" and how they are di%%erent. Answer: 6oth are scheduling tools used in pro)ect management" and both break down a pro)ect into the steps to be per%ormed and show the time needed %or each step. The $ERT chart shows the necessary se7uence o% acti#ities and identi%ies the critical path" which is the most time1 consuming set o% acti#ities. Thus the $ERT chart pro#ides e#en more in%ormation than the !antt chart. Eplanation: $ro)ect managers may need to reassign workers and e7uipment to speed up late acti#ities and stay on schedule. $age Re%: 1*< 'i%%iculty: 'i%%icult AA.-6: Analytic skills (b)ecti#e: *.8 +earning (utcome: 'iscuss the %actors that in%luence decisions about organi,ational structure -kill: .oncept 1/3) FedE and the U.-. $ostal -er#ice compete %or many o% the same customers. :ow are their strategies %or attracting customers di%%erentG Answer: FedE emphasi,es dependability and a speci%ic deli#ery time. The U.-. $ostal -er#ice doesn4t speci%y a deli#ery date %or %irst1class mail" but U.-. mail is less epensi#e. FedE tends to %ocus more on business customers than does the postal ser#ice. The postal ser#ice is more accessible" with daily deli#eries to homes. 0n general" the postal ser#ice pursues a lower1cost strategy. Eplanation: ?ith products such as priority mail and epress mail" the postal ser#ice is increasingly emphasi,ing dependable deli#ery. $age Re%: 1*1 'i%%iculty: 'i%%icult AA.-6: Analytic skills (b)ecti#e: *.3 +earning (utcome: 'iscuss the %actors that in%luence decisions about organi,ational structure -kill: .oncept 52 .opyright 9 /213 $earson Education" 0nc. $ublishing as $rentice :all 1/5) 'escribe three ways that >c'onald4s restaurants create time utility %or customers. Answer: >c'onald4s ser#es its %ood 7uickly" according to customer wishes. >any outlets ser#e break%ast during the morning hours. 'ri#e1through windows allow customers to order without ha#ing to park and enter the restaurant. >any outlets are open late to ser#e customers. Eplanation: A business creates time utility when it pro#ides products when consumers want them. $age Re%: 1&* 'i%%iculty: 'i%%icult (b)ecti#e: *./ +earning (utcome: 'iscuss the %actors that in%luence decisions about organi,ational structure -kill: .oncept 1/8) -uppose that economists are %orecasting a short economic recession %ollowed by strong economic growth. :ow might a large manu%acturing company ad)ust its capacity planning to respond to this %orecastG Answer: The company will probably wish to reduce its capacity in the short run" but retain its ability to increase capacity when the economy strengthens. .ompany eecuti#es may decide to reduce worker hours temporarily instead o% laying o%% workers. They may decide to idle some %acilities but keep them a#ailable to resume operations when economic growth picks up. Eplanation: A %irm4s capacity depends on how many people it employs and the number and si,e o% its %acilities. $age Re%: 1*5 'i%%iculty: 'i%%icult AA.-6: Analytic skills (b)ecti#e: *.5 +earning (utcome: 'escribe the skills and %unctions o% management -kill: -ynthesis 1/&) 0n recent years" some automakers ha#e increased manu%acturing operations in the southeastern United -tates. -uggest two reasons why they ha#e taken this step. Answer: The cost o% li#ing tends to be lower in this area" allowing companies to o%%er lower wages. There is also less union presence than in most other areas o% the country. -ome state go#ernments ha#e adopted policies" such as ta breaks" to encourage manu%acturing in their states. Eplanation: 6ecause location a%%ects production costs and %leibility" sound location planning is crucial %or %actories" o%%ices" and stores. $age Re%: 1*5 'i%%iculty: 'i%%icult AA.-6: Analytic skills (b)ecti#e: *.5 +earning (utcome: 'iscuss the %actors that in%luence decisions about organi,ational structure -kill: Application 51 .opyright 9 /213 $earson Education" 0nc. $ublishing as $rentice :all 1/*) (utsourcing has led to increasingly global supply chains. 'escribe three ways that a more global supply chain might be risky %or a company that outsources. Answer: ?hile outsourcing has usually been pro%itable" it has some risks. !lobal supply chains are more #ulnerable to natural disasters" such as the earth7uake and tsunami in Aapan and %looding in Thailand. $olitical unrest in other countries is also a risk. -ome countries ha#e nationali,ed businesses without warning to the companies. 0t is possible that U.-. lawmakers could impose laws discouraging outsourcing. .riminal acti#ities and e#en war could also disrupt supply chains. Eplanation: (utsourcing epands supply chains and increases their compleity. $age Re%: 1<&11<* 'i%%iculty: 'i%%icult AA.-6: 'ynamics o% the global economy (b)ecti#e: *.< +earning (utcome: Eplain the bene%its and challenges o% engaging in international business -kill: .oncept Alan Aackson is the %ounder and owner o% -ky -cooters" a small manu%acturing company located in -an Aose" .ali%ornia. For years" Alan toyed with ideas %or a motori,ed scooter. -eeing the success o% the -egway scooter" Alan decided to act. Although his design" the -ky .ruiser" is not as well1known as the -egway" it incorporates many o% the same bene%its. Adding to its appeal" the -ky .ruiser sells %or almost J1"222 less than the -egway. Firtually all -ky .ruisers are sold through direct appeals on the ;F. shopping network. (#erwhelmed by recent demand" Alan has hired his sister" .olette. .olette" a recent business school graduate" has suggested that Alan consider a )ust1in1time production system. Alan wonders i% this would be in his best interest. 1/<) ?ould a make1to1stock or make1to1order operations process be most e%%icient %or the production o% -ky .ruisersG Eplain. Answer: A make1to1stock operation would allow -ky -cooters to produce scooters with least epense and make them a#ailable %or immediate shipment. Eplanation: >ake1to1order operations are generally more time1consuming and more epensi#e. $age Re%: 1&= 'i%%iculty: >oderate AA.-6: Analytic skills (b)ecti#e: *./ +earning (utcome: 'iscuss the %actors that in%luence decisions about organi,ational structure -kill: Application 5/ .opyright 9 /213 $earson Education" 0nc. $ublishing as $rentice :all 1/=) ?hat ad#antages could -ky -cooters epect %rom switching to a )ust1in1time production systemG Answer: Aust1in1time systems sa#e money by reducing in#entories and replacing stop1and1go production with smooth mo#ement" making disruptions more #isible. Finding and eliminating disruptions by continuous impro#ement o% production is a ma)or ob)ecti#e o% A0T. Eplanation: Aust1in1time production is an eample o% a lean production system. $age Re%: 1<2 'i%%iculty: >oderate AA.-6: Analytic skills (b)ecti#e: *.& +earning (utcome: 'iscuss the %actors that in%luence decisions about organi,ational structure -kill: Application 132) Are there any potential negati#e conse7uences o% switching to a )ust1in1time production systemG Eplain. Answer: A0T production could ha#e negati#e conse7uences i% not well planned. -ince the re7uired materials arri#e )ust as they are needed" poor planning could shut down the whole production line. -imilarly" %ailure by any o% the members o% the supply chain could erase the bene%it o% the A0T strategy. Alan should care%ully plan the implementation o% a A0T production system. Eplanation: A0T production reduces costs when implemented properly" but it re7uires more o#ersight than a traditional production system. $age Re%: 1<2 'i%%iculty: 'i%%icult AA.-6: Analytic skills (b)ecti#e: *.& +earning (utcome: 'iscuss the %actors that in%luence decisions about organi,ational structure -kill: Application 131) !i#en the price o% the -ky .ruiser" what is a reasonable goal %or product 7ualityG Answer: The -ky .ruiser is priced about J1"222 less than its competitor" -egway. The -ky .ruiser should be sa%e and per%orm e%%iciently" but it need not eceed the per%ormance o% -egway. A reasonable goal is to per%orm close to the -egway standard. The -ky .ruiser will probably ha#e %ew CetrasC and its appearance will likely be %unctional rather than sleek. Eplanation: (%%ering a lower1cost product is one strategy that -ky -cooters will use to attract customers. $age Re%: 1<1 'i%%iculty: 'i%%icult AA.-6: Analytic skills (b)ecti#e: *.* +earning (utcome: 'escribe the skills and %unctions o% management -kill: Application 53 .opyright 9 /213 $earson Education" 0nc. $ublishing as $rentice :all 13/) ?hat are three possible changes that Alan Aackson can make in his company to emphasi,e 7uality impro#ementG Answer: $ossible approaches include %orming a 7uality management team" encouraging 7uality ownership" adopting total 7uality management" and creating a team to recei#e and re#iew input %rom customers and potential customers. Eplanation: 0n addition to controlling 7uality" businesses seek to build 7uality into goods and ser#ices. $age Re%: 1<311<5 'i%%iculty: 'i%%icult AA.-6: Re%lecti#e thinking skills (b)ecti#e: *.* +earning (utcome: 'escribe the skills and %unctions o% management -kill: Application 133) 0% a A0T production system is implemented at -ky -cooters" how might the %irm4s supply chain strategy be a%%ectedG Answer: Alan and .olette must coordinate -ky -cooters4 acti#ities with a network o% other %irms. They will need to regard these %irms as members o% a coordinated supply system. A A0T production system will be challenging since timing will be o% utmost importance. -ky -cooters will need to communicate 7uickly and e%%ecti#ely with its partners and may need the latest technologies in order to implement the more challenging A0T system. Eplanation: -upply chain strategy is based on the idea that members o% the chain will gain competiti#e ad#antage by working as a coordinated unit. Although each company looks out %or its own interests" it works closely with suppliers and customers throughout the chain. $age Re%: 1<2" 1<& 'i%%iculty: 'i%%icult AA.-6: Analytic skills (b)ecti#e: *.< +earning (utcome: 'iscuss the %actors that in%luence decisions about organi,ational structure -kill: Application 55 .opyright 9 /213 $earson Education" 0nc. $ublishing as $rentice :all EKTrade is a retail organi,ation that owns specialty stores %or a di#erse range o% product categories" ranging %rom home %urnishings and sporting goods to books and gourmet %oods. ?ith more than 82 stores" the %irm is now considering entering the #ery lucrati#e low1cost market by launching .lotheLa chain o% retail stores that o%%ers a%%ordable %ashion %or men" women" and children. 135) ?hich o% the %ollowing" i% true" would strengthen the argument %or being a low1cost playerG A) .ustomers who belie#e that they need high le#els o% customer ser#ice are more likely to choose a niche retailer. 6) An epected upturn in the economy will increase consumer purchasing power. .) 6eing the low1cost player re7uires a company to eamine e#ery aspect o% its supply chain in order to identi%y opportunities to impro#e e%%iciency. ') EKTrade has a longer history selling clothes %or men and women than it has selling clothes %or children. E) The retailer4s logistics operations are more e%%icient than the industry norm. Answer: E Eplanation: E) .ompeting on price isn4t easy. Hot e#ery organi,ation can do it. :owe#er" organi,ations that are already e%%icient are better suited to try to win as a low1cost player. -o .hoice E strengthens the argument. .hoices A" 6" and . weaken the argument by suggesting problems with the low1cost approach. .hoice ': The length o% acti#ity in di%%erent product lines doesn4t help resol#e the 7uestion o% whether low1cost is the way to go. $age Re%: 1&& 'i%%iculty: >oderate AA.-6: Re%lecti#e thinking skills (b)ecti#e: *.1 -kill: .ritical Thinking 58 .opyright 9 /213 $earson Education" 0nc. $ublishing as $rentice :all -tardust 4n4 .lay is a manu%acturer and mass marketer o% personal care products and nutritional supplements. The company plans to launch a line o% organic beauty products that are made entirely %rom mineral pigments and organic plant etracts. These products" though epensi#e to process" o%%er much higher margins than the company4s current lines o% beauty products. The market %or organic beauty products is relati#ely untapped" with )ust a hand%ul o% players. 138) ?hich o% the %ollowing" i% true" would strengthen the case %or -tardust 4n4 .lay to begin de#eloping organic beauty productsG A) Retailers report a consumer pre%erence %or inepensi#e skin1care and beauty products that do a better )ob than the ones currently a#ailable. 6) (rganic raw materials sourced %rom economically weak countries pro#ide a source o% li#elihood %or the local population. .) (rganic raw materials %or such products can be sourced %rom de#eloping countries at a lower cost. ') The %irst mo#er in this industry is witnessing reasonable growth. E) A growing number o% women 1<135 consider the health impact o% chemicals %ound in a beauty product when making a cosmetics purchase decision. Answer: E Eplanation: E) -tardust 4n4 .lay sees many potential ad#antages in marketing organic beauty products. Are there enough people interested in the product to make the higher production costs worthwhileG .hoice E suggests there are" indicating that young women@who comprise a large cosmetics market@care about the chemicals in their beauty products. .hoice A does not say much about organic products. Also" organic products are epensi#e to manu%acture and o%%er higher margins. 0n other words" they are more epensi#e than nonorganic beauty products. .hoice 6 eplains how a new -tardust 4n4 .lay line o% organic beauty products might bene%it people in economically weak countries" but it tells us nothing about whether the line would bene%it -tardust 4n4 .lay. .hoice . pro#ides general in%ormation about raw material sourcing %or the proposed line. This is not an absolute ad#antage %or -tardust 4n4 .lay" as the same conditions apply to competitors. .hoice ' only tells us that the %irst mo#er in the market has seen Creasonable growth.C This %irst mo#er might #ery well be a small niche marketer. -tardust 4n4 .lay is a mass marketer" so Creasonable growthC to a smaller company may be insigni%icant to -tardust 4n4 .lay. $age Re%: 1*2 'i%%iculty: >oderate AA.-6: Re%lecti#e thinking skills (b)ecti#e: *.3 -kill: .ritical Thinking 5& .opyright 9 /213 $earson Education" 0nc. $ublishing as $rentice :all 13&) ?hich o% the %ollowing" i% true" would weaken the argument %or de#eloping organic beauty productsG A) Few companies ha#e obtained patents %or their organic product inno#ations. 6) .ompetitors in this segment are 7uick to copy any new product inno#ation and mimic product design. .) (rganic o%%erings ha#e a shorter shel%1li%e compared to synthetic products. ') .ustomers need to be educated about the relati#e bene%its o%%ered by organic products. E) Financial analysis shows that it would take the company 1.8 years to reco#er its initial in#estment. Answer: 6 Eplanation: 6) 0% -tardust doubts whether it can achie#e and maintain a competiti#e ad#antage in the organic beauty products market" then the company is unlikely to pursue the proposed line. .hoice 6 points to a lack o% sustainable competiti#e ad#antage" as competitors can be epected to 7uickly copy -tardust4s product and promotions. .hoice 6 weakens the argument. .hoice A is a nonissue" as -tardust could use di%%erent processes and come up with di%%erent products. .hoice . doesn4t necessarily weaken the argument" as a shorter shel%1li%e suggests that consumers would need to repurchase the cosmetics more %re7uently. .hoice ' indicates a need %or increased customer awareness" a promotional challenge that would be true %or many inno#ati#e products. Alone" this isn4t enough. .hoice E tells us how long it would take the company to reco#er its in#estment" but without a %rame o% re%erence we don4t know i% this strengthens or weakens the argument. 0% it usually takes two or more years to reco#er an in#estment" then this strengthens the argument %or de#eloping an organic line. $age Re%: 1*2 'i%%iculty: >oderate AA.-6: Re%lecti#e thinking skills (b)ecti#e: *.3 -kill: .ritical Thinking 5* .opyright 9 /213 $earson Education" 0nc. $ublishing as $rentice :all Toys %or $ops is a small business that %ocuses on indoor and outdoor games intended %or an adult male clientele. 0t stocks a range o% %antasy action %igures and models" low1impact athletic accessories such as Frisbees and cro7uet sets" and a wide range o% #ideo games. The business has grown to the point that a larger location is needed. Recently" a location has come up %or rent at a nearby strip mall at a %a#orable rate. The %acility would pro#ide the business with much needed shel% space and more room %or employees behind the counter. :owe#er" the strip mall is in an area that has also seen a rapid increase in petty crime. The .E( %eels that the ad#antages o% the new location support the decision to relocate. 13*) ?hich o% the %ollowing" i% true" would weaken the .E(4s positionG A) The i%%y reputation o% the area in which the strip mall is located would make the business4s current employees uneasy about their sa%ety at work" which would lead to increased turno#er in sta%%. 6) The lower cost o% li#ing in the new neighborhood would make it easier to hire employees willing to accept lower salaries. .) Fideo games sales" a%ter se#eral years o% increases" ha#e reached a plateau and are unlikely to show %urther growth in the net %ew years. ') The strip mall has no other current #acancies. E) The current sta%% has been stable %or se#eral years and is well #ersed in details o% #ideo games and other specialty items. Answer: A Eplanation: A) Two o% the key human resource management responsibilities o% a small business are maintaining morale and protecting employees4 physical conditions. ?hen these issues are neglected" turno#er increases" leading to the need to hire and train new sta%% more o%ten than necessary. :ence .hoice A weakens the .E(4s position. .on#ersely" .hoice 6 might strengthen the decision" by showing that labor costs can be controlled through the mo#e. .hoice . might be a consideration i% the business sold only #ideo games" but in %act this is )ust one product line the %irm handles. .hoice '" i% anything" strengthens the .E(4s position by suggesting that the strip mall is being e%%ecti#ely managed by its owners. .hoice E deals with a strength o% the company in its eisting location" but does not address the wisdom o% the mo#e. $age Re%: 1*5 'i%%iculty: >oderate AA.-6: Re%lecti#e thinking skills (b)ecti#e: *.5 -kill: .ritical Thinking 5< .opyright 9 /213 $earson Education" 0nc. $ublishing as $rentice :all 13<) ?hich o% the %ollowing" i% true" might )usti%y the relocation as a prudent mo#e %rom the human resource point o% #iewG A) 'e#eloping a Facebook page made the business known to a large group o% potential customers li#ing in a much wider regional area. 6) Federal crime reports show that the greater metro area has seen an increase in car the%ts. .) The new location is located near a ma)or %reeway interchange. ') Fantasy action %igures are o%ten remaindered %or below wholesale cost because their popularity is short1li#ed. E) Anonymous notes in the %irm4s suggestion bo repeatedly cited cramped conditions and lack o% pri#acy as the most annoying %actors in working conditions at the present site. Answer: E Eplanation: E) ?hile the potential %or crime is troublesome" employees might consider the trade1o%% acceptable i% the mo#e results in a signi%icant impro#ement in working conditions. -o .hoice E is the strongest argument in %a#or o% the mo#e. .hoice A doesn4t address the human resource impact o% relocating to a marginal neighborhood. .hoices 6 and ' ha#e no rele#ance to the proposed mo#e. .hoice . might support the mo#e by suggesting that the location would be an easier commute %or employees" but it4s also possible that the interchange area would be a tra%%ic bottleneck. $age Re%: 1*5 'i%%iculty: >oderate AA.-6: Re%lecti#e thinking skills (b)ecti#e: *.5 -kill: .ritical Thinking 5= .opyright 9 /213 $earson Education" 0nc. $ublishing as $rentice :all .lean -weep .orporation is a consumer goods company manu%acturing home1care cleaning products. -ales o% -parkle" its %lagship disin%ectant" and some o% its other cleaning products ha#e been sluggish during the last two 7uarters. (% late" retailers ha#e also shown a lack o% con%idence in these product lines. The management is looking at di%%erent ways to impro#e the e%%iciency o% its entire supply chain. -ome senior eecuti#es belie#e that the company should shi%t to a )ust1in1 time logistics system. :owe#er" this opinion does not ha#e unanimous support. 13=) ?hich o% the %ollowing" i% true" would weaken the argument %or the shi%t to a )ust1in1time logistics systemG A) .arrying too much in#entory results in higher1than1necessary in#entory1carrying costs. 6) Under the new system" the company would ha#e in#entory su%%icient %or %ewer days o% operations. .) .lean -weep4s most important suppliers pre%er a %leible supply schedule. ') 0mplementing the system would re7uire an etensi#e o#erhaul o% the eisting supply chain. E) 'emand %or .lean -weep4s products is stable and predictable. Answer: ' Eplanation: ') .hoice ' weakens the argument since the bene%its o% a )ust1in1time system could be outweighed by the costs o% re1engineering the entire supply chain. .hoice A: A )ust1in1 time system is designed to lower in#entory1carrying costs" so .hoice A would strengthen the argument. .hoice 6 describes the system accurately but pro#ides no reason not to adopt it. .hoice . would strengthen the argument by suggesting that the transition could be easy. .hoice E strengthens the argument by suggesting that it will be possible to operate with lower in#entory le#els. $age Re%: 1<2 'i%%iculty: >oderate AA.-6: Re%lecti#e thinking skills (b)ecti#e: *.& -kill: .ritical Thinking 82 .opyright 9 /213 $earson Education" 0nc. $ublishing as $rentice :all 152) ?hich o% the %ollowing" i% true" would strengthen the argument %or the shi%t to a )ust1in1time in#entory systemG A) .lean -weep4s storage and handling costs are signi%icant. 6) .lean -weep4s suppliers ha#e not adopted )ust1in1time systems. .) The current system re7uires higher in#entory le#els during the summer as compared to the winter. ') 'emand %or -parkle #aries depending on the promotional strategies o% .lean -weep4s retail partners. E) -parkle comes in many #arieties" none o% which is more popular than the leading brand. Answer: A Eplanation: A) Aust1in1time logistics systems can reduce handling and storage costs" which is an ad#antage when those costs are signi%icant" .hoice A. .hoice 6" i% anything" weakens the argument" as the system might be e#en more e%%ecti#e i% others in the supply chain used similar systems. .hoice . does not point out a %law in the current system" as it is reasonable that people would use more cleaning products in warmer weather. .hoice ' suggests that demand is hard to predict" which would weaken the argument. .hoice E: -parkle4s position in the market does not tell which in#entory system makes the most sense. $age Re%: 1<2 'i%%iculty: >oderate AA.-6: Re%lecti#e thinking skills (b)ecti#e: *.& -kill: .ritical Thinking 151) ?hich o% the %ollowing 7uestions is +EA-T rele#ant to the issue o% whether the company should switch to a )ust1in1time logistics systemG A) 0s demand %or .lean -weep4s products predictableG 6) .an suppliers assure %ast" %re7uent" and %leible deli#eryG .) 0s technology a#ailable to manage the logisticsG ') 'o end users currently %a#or making the transitionG E) 'oes .lean -weep ha#e access to accurate in%ormation about customer demandG Answer: ' Eplanation: ') All the choices speak to re7uirement %or a )ust1in1time system ecept .hoice '. A success%ul )ust1in1time system would be in#isible to most customers. They would simply ha#e access to the product when they wanted it. Also" customers who buy cleaning supplies would not ha#e an in%ormed opinion o% the company4s supply chain operation" so their initial opinion is not crucial here. $age Re%: 1<2 'i%%iculty: 'i%%icult AA.-6: Re%lecti#e thinking skills (b)ecti#e: *.& -kill: .ritical Thinking 81 .opyright 9 /213 $earson Education" 0nc. $ublishing as $rentice :all 15/) A senior eecuti#e claims that a mo#e to a C)ust1in1timeC logistics system would be a mistake because e#en i% the system is e%%ecti#e at reducing in#entory and handling costs" the change would not be noticed by customers. The eecuti#e claims that the change should be re)ected as it runs counter to .lean -weep4s goal o% being Ccustomer1%ocused.C ?hich o% the %ollowing is the best criticism o% the senior eecuti#e4s positionG A) 0t does not pro#e that %ocusing on the customer helps an organi,ation become success%ul. 6) 0t does not suggest an impro#ement to .lean -weep4s current system. .) 0t %ails to recogni,e that ha#ing a more e%%icient supply chain will allow .lean -weep to o%%er more #alue to its customers. ') 0t does not establish that the costs o% switching to the new system would outweigh any cost1 reduction bene%its the system might pro#ide. E) 0t does not describe the in#entory systems used by .lean -weep4s competitors. Answer: . Eplanation: .) The senior eecuti#e is basically saying that any change would ha#e to be noticed by customers in order %or the change to be a good one. 6ut that doesn4t make sense. All other things being e7ual" sa#ing money in production and distribution is a good thing" e#en i% the customers don4t notice the changes. 6esides" as .hoice . points out" ha#ing lower costs can allow .lean -weep to o%%er more #alue" %or eample by lowering prices. .hoice A: The #alue o% being customer1%ocused isn4t at issue here. The problem is that the senior eecuti#e takes it too %ar in claiming that e#ery good idea must be percei#ed by the customer. .hoices 6" '" and E are accurate in that the argument does not do these things" but there is no reason to belie#e that they are necessary. $age Re%: 1<2 'i%%iculty: 'i%%icult AA.-6: Re%lecti#e thinking skills (b)ecti#e: *.& -kill: .ritical Thinking 8/ .opyright 9 /213 $earson Education" 0nc. $ublishing as $rentice :all A so%tware de#elopment company is designing an e#aluation plan %or its so%tware programmers. The company %eels that changes are necessary because it lacks the %acts it needs to distinguish outstanding so%tware programmers %rom those that are only a#erage" or worse. $re#iously" the company paid so%tware programmers a %lat salary and based e#aluations on super#isors4 opinions. How" howe#er" the company is considering the %ollowing measures %or its so%tware programmers: >easurement -trategy Alpha: -o%tware programmers will be e#aluated based on the total number o% lines o% code that they produce. >easurement -trategy 6eta: -o%tware programmers will be e#aluated based on their ability to produce computer code that is %ree o% errors. >easurement -trategy !amma: -o%tware programmers will be e#aluated based on the market success o% the products they produce. 153) ?hich o% the %ollowing" i% true" would weaken the argument that the company should use >easurement -trategy AlphaG A) -o%tware programmers usually ha#e no direct contact with customers and do not determine product speci%ications. 6) The programming languages used by the company re7uire %ewer lines o% code to per%orm a %unction than the programming languages used by other companies. .) Any so%tware programmer can choose to increase the number o% lines o% code it takes to per%orm any %unction. ') An e%%ecti#e so%tware programming team is essential %or the %inancial success o% the company. E) >any o% the current so%tware programmers would not recei#e strong e#aluations i% the company used >easurement -trategy Alpha. Answer: . Eplanation: .) >easurement -trategy Alpha rewards so%tware programmers based on the number o% lines o% code that they produce. 6ut i% .hoice . is true then the so%tware programmers can arti%icially in%late the number o% lines it takes to do something. They could abuse this approach by taking thousands o% lines o% code to do simple things. They would seem producti#e according to >easurement -trategy Alpha" but they really wouldn4t be. .hoice A is sort o% rele#ant to >easurement -trategy !amma" not Alpha. .hoice 6 is a reason why >easurement -trategy Alpha couldn4t be used to compare programmers %rom di%%erent companies" but these strategies concern the company4s employees only. .hoice ' suggests that getting this right is important but does not tell us whether Alpha will help the company get it right. .hoice E would be a problem i% we knew that all o% the current so%tware programmers are ecellent" but the whole point o% the measurement plan is that we don4t know who is doing a good )ob. -o the %act that Alpha would be bad %or some employees doesn4t suggest that there is anything wrong with Alpha. $age Re%: 1<2 'i%%iculty: >oderate AA.-6: Re%lecti#e thinking skills (b)ecti#e: *.& -kill: .ritical Thinking 83 .opyright 9 /213 $earson Education" 0nc. $ublishing as $rentice :all 155) A decision to use >easurement -trategy 6eta would assume which o% the %ollowingG A) -o%tware errors are more common than hardware errors. 6) -o%tware errors a%%ect all users e7ually. .) The impact o% so%tware errors has been greater recently. ') ?hen a so%tware error occurs" its e%%ects are always clear to the %inal user o% the product. E) ?hen a so%tware error is disco#ered" it is possible to determine who is responsible %or it. Answer: E Eplanation: E) >easurement -trategy 6eta punishes people %or their errors" but i% .hoice E isn4t true" then you can4t tell who made the error. .hoice E would ha#e to be true in order %or >easurement -trategy 6eta to work" and so it must be assumed. .hoice A: This is about so%tware" not hardware" so nothing about hardware errors need be assumed. .hoice 6 could be true" and probably isn4t" but it doesn4t need to be true in order %or 6eta to work. .hoices . and ' would strengthen the case %or 6eta i% they were true" but they don4t need to be true in order %or 6eta to work. $age Re%: 1<2 'i%%iculty: 'i%%icult AA.-6: Re%lecti#e thinking skills (b)ecti#e: *.& -kill: .ritical Thinking 158) The choice to use only >easurement -trategy 6eta would be #ulnerable to criticism because it would gi#e the so%tware programmers an incenti#e to do whatG A) make o#erly optimistic commitments to customer1%acing employees at the company 6) spend too much time checking their code in order to eliminate any possibility o% error .) de#elop pro)ects that meet programming re7uirements but not market re7uirements ') skip the 7uality assurance checks that re#eal the most damaging so%tware errors E) produce more lines o% code than is re7uired to per%orm a gi#en %unction Answer: 6 Eplanation: 6) A#oiding errors is good" but so is %inishing on time. 0% the %ocus is only on a#oiding errors" then the so%tware programmers will spend way too much time a#oiding any chance that there is an error in their code somewhere. .hoice 6 describes this issue. .hoices A and . are rele#ant to !amma. .hoice ' would be sol#ed by 6eta. .hoice E is a problem with Alpha. $age Re%: 1<2 'i%%iculty: >oderate AA.-6: Re%lecti#e thinking skills (b)ecti#e: *.& -kill: .ritical Thinking 85 .opyright 9 /213 $earson Education" 0nc. $ublishing as $rentice :all 15&) The choice to use only >easurement -trategy !amma would be #ulnerable to criticism in what wayG A) >any %actors other than so%tware programming are re7uired %or market success. 6) 0t sets up a goal that is unattainable. .) 0t underestimates the e%%ect that good so%tware programming can ha#e on the customer4s eperience. ') 0t would re7uire the company to change its current approach to e#aluating so%tware programmers. E) 0t %ails to establish a goal that re7uires any signi%icant e%%ort to achie#e. Answer: A Eplanation: A) 0t makes sense i% the programmers are rewarded when the products do well" but i% they are e#aluated solely on that measure" then the results might wind up being arbitrary. As .hoice A points out" e#en i% the programmers do their )ob well" many things need to happen be%ore the product becomes a success. (ne principle o% moti#ation is that i% you are going to reward people when a goal is achie#ed" then those people should be able to make that goal happen. >easurement -trategy !amma ignores this and is #ulnerable to the criticism in .hoice A. .hoices 6 and E: There is no reason to belie#e that market success is impossible B.hoice 6) or too easy B.hoice E). .hoice . has it backwards. 0% anything" !amma o#erestimates the in%luence o% so%tware programming. .hoice ' is accurate but is not a problem. The whole point is to change the system. $age Re%: 1<2 'i%%iculty: 'i%%icult AA.-6: Re%lecti#e thinking skills (b)ecti#e: *.& -kill: .ritical Thinking 15*) ?hich o% the %ollowing would be most similar to a decision to use >easurement -trategy Alpha and >easurement -trategy 6eta togetherG A) Aiom e#aluates its sales sta%% on the number o% sales they make and the si,e o% those sales. 6) $ersonal 'e#ices rewards its brand managers more hea#ily %or sales o% new products than %or sales o% pre#iously eisting products. .) -tardust 4n4 .lay rewards production line workers based on meeting production 7uotas and a#oiding product de%ects. ') (ak >eteors customer ser#ice agents are e#aluated based on the 7uantity o% calls they answer and the number o% %ollow1up purchases that those callers make. E) -pa ?orld #alet parkers get a yearly bonus based on o#erall customer per%ormance and supplement their income with tips %rom customers. Answer: . Eplanation: .) Alpha rewards based on output" and 6eta rewards based on a#oiding errors. .hoice . matches this by rewarding based on 7uantity and a#oiding de%ects. Hone o% the other choices has these components. $age Re%: 1<2 'i%%iculty: 'i%%icult AA.-6: Re%lecti#e thinking skills (b)ecti#e: *.& -kill: .ritical Thinking 88 .opyright 9 /213 $earson Education" 0nc. $ublishing as $rentice :all 15<) >anagement has decided to combine all three >easurement -trategies in order to gi#e the so%tware programmers incenti#es that match the company4s goals. 0% the products meet their !amma goals" so%tware programmers who ha#e met either their Alpha goals or their 6eta goals will be rewarded" but i% the products %ail to meet their !amma goals" so%tware programmers will not be rewarded unless they ha#e met both their Alpha goals and their 6eta goals. Under this system" which o% the %ollowing .AHH(T be trueG A) A so%tware programmer who has met his or her Alpha goals is not rewarded. 6) A so%tware programmer who has not met his or her 6eta goals is rewarded. .) A so%tware programmer who has met neither his or her Alpha goals nor his or her 6eta goals is not rewarded. ') The company meets its !amma goals" and a so%tware programmer who meets his or her Alpha goals is not rewarded. E) The company does not meet its !amma goals" and a so%tware programmer who meets his or her 6eta goals is not rewarded. Answer: ' Eplanation: ') 0% the company hits its !amma goals" then the so%tware programmers need to hit )ust one o% Alpha and 6eta in order to be rewarded. -o .hoice ' is impossible. .hoice A could be true i% this person did not meet his or her Alpha goals and the company did not meet its !amma goals. .hoice 6 could be true i% this person met his or her Alpha goals and the company met its !amma goals. .hoice .: Those who %ail both goals aren4t guaranteed a reward. .hoice E could be true i% the person did not meet his or her Alpha goals. $age Re%: 1<2 'i%%iculty: 'i%%icult AA.-6: Re%lecti#e thinking skills (b)ecti#e: *.& -kill: .ritical Thinking 8& .opyright 9 /213 $earson Education" 0nc. $ublishing as $rentice :all